Kronecker Delta Expressions











up vote
2
down vote

favorite












I am trying to understand the Kronecker Delta and want to clarify. Considering the definition of the Kronecker Delta and assuming $i=j=k$ for the following situations:



I know that $delta _j^i delta _i^j $ is equal to $N$ where $N$ is the dimension of space. Would this mean that $delta _j^i delta _k^j delta _i^k $ would also be equal to $N$?



Similarly, since $delta _i^i delta _j^j $ is equal to $N^2$, would $delta _i^i delta _j^j delta _k^k $ be equal to $N^3$ ?



Last, is $delta _i^j delta _j^k $ just equal to $delta _i^k = 1 $ ?










share|cite|improve this question
























  • @AlvinJin This should be an answer not a comment
    – Eddy
    Sep 25 at 20:51










  • If $i=k$, wouldn't the $delta_i^k$ be equal to 1, by definition though?
    – Cave Johnson
    Sep 25 at 20:53















up vote
2
down vote

favorite












I am trying to understand the Kronecker Delta and want to clarify. Considering the definition of the Kronecker Delta and assuming $i=j=k$ for the following situations:



I know that $delta _j^i delta _i^j $ is equal to $N$ where $N$ is the dimension of space. Would this mean that $delta _j^i delta _k^j delta _i^k $ would also be equal to $N$?



Similarly, since $delta _i^i delta _j^j $ is equal to $N^2$, would $delta _i^i delta _j^j delta _k^k $ be equal to $N^3$ ?



Last, is $delta _i^j delta _j^k $ just equal to $delta _i^k = 1 $ ?










share|cite|improve this question
























  • @AlvinJin This should be an answer not a comment
    – Eddy
    Sep 25 at 20:51










  • If $i=k$, wouldn't the $delta_i^k$ be equal to 1, by definition though?
    – Cave Johnson
    Sep 25 at 20:53













up vote
2
down vote

favorite









up vote
2
down vote

favorite











I am trying to understand the Kronecker Delta and want to clarify. Considering the definition of the Kronecker Delta and assuming $i=j=k$ for the following situations:



I know that $delta _j^i delta _i^j $ is equal to $N$ where $N$ is the dimension of space. Would this mean that $delta _j^i delta _k^j delta _i^k $ would also be equal to $N$?



Similarly, since $delta _i^i delta _j^j $ is equal to $N^2$, would $delta _i^i delta _j^j delta _k^k $ be equal to $N^3$ ?



Last, is $delta _i^j delta _j^k $ just equal to $delta _i^k = 1 $ ?










share|cite|improve this question















I am trying to understand the Kronecker Delta and want to clarify. Considering the definition of the Kronecker Delta and assuming $i=j=k$ for the following situations:



I know that $delta _j^i delta _i^j $ is equal to $N$ where $N$ is the dimension of space. Would this mean that $delta _j^i delta _k^j delta _i^k $ would also be equal to $N$?



Similarly, since $delta _i^i delta _j^j $ is equal to $N^2$, would $delta _i^i delta _j^j delta _k^k $ be equal to $N^3$ ?



Last, is $delta _i^j delta _j^k $ just equal to $delta _i^k = 1 $ ?







vectors tensors kronecker-symbol kronecker-delta






share|cite|improve this question















share|cite|improve this question













share|cite|improve this question




share|cite|improve this question








edited Sep 25 at 20:41

























asked Sep 25 at 20:10









Cave Johnson

206




206












  • @AlvinJin This should be an answer not a comment
    – Eddy
    Sep 25 at 20:51










  • If $i=k$, wouldn't the $delta_i^k$ be equal to 1, by definition though?
    – Cave Johnson
    Sep 25 at 20:53


















  • @AlvinJin This should be an answer not a comment
    – Eddy
    Sep 25 at 20:51










  • If $i=k$, wouldn't the $delta_i^k$ be equal to 1, by definition though?
    – Cave Johnson
    Sep 25 at 20:53
















@AlvinJin This should be an answer not a comment
– Eddy
Sep 25 at 20:51




@AlvinJin This should be an answer not a comment
– Eddy
Sep 25 at 20:51












If $i=k$, wouldn't the $delta_i^k$ be equal to 1, by definition though?
– Cave Johnson
Sep 25 at 20:53




If $i=k$, wouldn't the $delta_i^k$ be equal to 1, by definition though?
– Cave Johnson
Sep 25 at 20:53










1 Answer
1






active

oldest

votes

















up vote
1
down vote



accepted










It looks to me like you're inherently using the Einstein summation convention, familiar in general relativity. For the purposes of this question, this means that any indices seen twice are summed over.



When you say $delta^i_j delta^j_i = N$, for example, this implicitly means $displaystyle sum_{i=1}^N displaystyle sum_{j=1}^N delta^i_j delta^j_i = N$ which is indeed true since the sum is non-vanishing whenever $i=j$ and this happens $N$ times.



Similarly, $delta^i_j delta^j_k delta^k_i = displaystyle sum_{i=1}^N displaystyle sum_{j=1}^N displaystyle sum_{k=1}^N delta^i_j delta^j_k delta^k_i = N$, as you surmise.



Also $delta^i_i delta^j_j delta^k_k = displaystyle sum_{i=1}^N displaystyle sum_{j=1}^N displaystyle sum_{k=1}^N delta^i_i delta^j_j delta^k_k = N^3$, as you suspected.



Finally, $delta^j_i delta^k_j = delta^k_i$ but this is not a scalar quantity ($neq 1$). Instead it has a separate value for each index $i$ and $k$. You can think of it as being represented by a matrix whose $(i,k)$th entry is $1$ if $i=k$ and $0$ otherwise (identity matrix in $N$ dimensions).



This is a bit of a simplification of the whole picture but the essence of the machinery is here.






share|cite|improve this answer





















    Your Answer





    StackExchange.ifUsing("editor", function () {
    return StackExchange.using("mathjaxEditing", function () {
    StackExchange.MarkdownEditor.creationCallbacks.add(function (editor, postfix) {
    StackExchange.mathjaxEditing.prepareWmdForMathJax(editor, postfix, [["$", "$"], ["\\(","\\)"]]);
    });
    });
    }, "mathjax-editing");

    StackExchange.ready(function() {
    var channelOptions = {
    tags: "".split(" "),
    id: "69"
    };
    initTagRenderer("".split(" "), "".split(" "), channelOptions);

    StackExchange.using("externalEditor", function() {
    // Have to fire editor after snippets, if snippets enabled
    if (StackExchange.settings.snippets.snippetsEnabled) {
    StackExchange.using("snippets", function() {
    createEditor();
    });
    }
    else {
    createEditor();
    }
    });

    function createEditor() {
    StackExchange.prepareEditor({
    heartbeatType: 'answer',
    convertImagesToLinks: true,
    noModals: true,
    showLowRepImageUploadWarning: true,
    reputationToPostImages: 10,
    bindNavPrevention: true,
    postfix: "",
    imageUploader: {
    brandingHtml: "Powered by u003ca class="icon-imgur-white" href="https://imgur.com/"u003eu003c/au003e",
    contentPolicyHtml: "User contributions licensed under u003ca href="https://creativecommons.org/licenses/by-sa/3.0/"u003ecc by-sa 3.0 with attribution requiredu003c/au003e u003ca href="https://stackoverflow.com/legal/content-policy"u003e(content policy)u003c/au003e",
    allowUrls: true
    },
    noCode: true, onDemand: true,
    discardSelector: ".discard-answer"
    ,immediatelyShowMarkdownHelp:true
    });


    }
    });














    draft saved

    draft discarded


















    StackExchange.ready(
    function () {
    StackExchange.openid.initPostLogin('.new-post-login', 'https%3a%2f%2fmath.stackexchange.com%2fquestions%2f2930727%2fkronecker-delta-expressions%23new-answer', 'question_page');
    }
    );

    Post as a guest















    Required, but never shown

























    1 Answer
    1






    active

    oldest

    votes








    1 Answer
    1






    active

    oldest

    votes









    active

    oldest

    votes






    active

    oldest

    votes








    up vote
    1
    down vote



    accepted










    It looks to me like you're inherently using the Einstein summation convention, familiar in general relativity. For the purposes of this question, this means that any indices seen twice are summed over.



    When you say $delta^i_j delta^j_i = N$, for example, this implicitly means $displaystyle sum_{i=1}^N displaystyle sum_{j=1}^N delta^i_j delta^j_i = N$ which is indeed true since the sum is non-vanishing whenever $i=j$ and this happens $N$ times.



    Similarly, $delta^i_j delta^j_k delta^k_i = displaystyle sum_{i=1}^N displaystyle sum_{j=1}^N displaystyle sum_{k=1}^N delta^i_j delta^j_k delta^k_i = N$, as you surmise.



    Also $delta^i_i delta^j_j delta^k_k = displaystyle sum_{i=1}^N displaystyle sum_{j=1}^N displaystyle sum_{k=1}^N delta^i_i delta^j_j delta^k_k = N^3$, as you suspected.



    Finally, $delta^j_i delta^k_j = delta^k_i$ but this is not a scalar quantity ($neq 1$). Instead it has a separate value for each index $i$ and $k$. You can think of it as being represented by a matrix whose $(i,k)$th entry is $1$ if $i=k$ and $0$ otherwise (identity matrix in $N$ dimensions).



    This is a bit of a simplification of the whole picture but the essence of the machinery is here.






    share|cite|improve this answer

























      up vote
      1
      down vote



      accepted










      It looks to me like you're inherently using the Einstein summation convention, familiar in general relativity. For the purposes of this question, this means that any indices seen twice are summed over.



      When you say $delta^i_j delta^j_i = N$, for example, this implicitly means $displaystyle sum_{i=1}^N displaystyle sum_{j=1}^N delta^i_j delta^j_i = N$ which is indeed true since the sum is non-vanishing whenever $i=j$ and this happens $N$ times.



      Similarly, $delta^i_j delta^j_k delta^k_i = displaystyle sum_{i=1}^N displaystyle sum_{j=1}^N displaystyle sum_{k=1}^N delta^i_j delta^j_k delta^k_i = N$, as you surmise.



      Also $delta^i_i delta^j_j delta^k_k = displaystyle sum_{i=1}^N displaystyle sum_{j=1}^N displaystyle sum_{k=1}^N delta^i_i delta^j_j delta^k_k = N^3$, as you suspected.



      Finally, $delta^j_i delta^k_j = delta^k_i$ but this is not a scalar quantity ($neq 1$). Instead it has a separate value for each index $i$ and $k$. You can think of it as being represented by a matrix whose $(i,k)$th entry is $1$ if $i=k$ and $0$ otherwise (identity matrix in $N$ dimensions).



      This is a bit of a simplification of the whole picture but the essence of the machinery is here.






      share|cite|improve this answer























        up vote
        1
        down vote



        accepted







        up vote
        1
        down vote



        accepted






        It looks to me like you're inherently using the Einstein summation convention, familiar in general relativity. For the purposes of this question, this means that any indices seen twice are summed over.



        When you say $delta^i_j delta^j_i = N$, for example, this implicitly means $displaystyle sum_{i=1}^N displaystyle sum_{j=1}^N delta^i_j delta^j_i = N$ which is indeed true since the sum is non-vanishing whenever $i=j$ and this happens $N$ times.



        Similarly, $delta^i_j delta^j_k delta^k_i = displaystyle sum_{i=1}^N displaystyle sum_{j=1}^N displaystyle sum_{k=1}^N delta^i_j delta^j_k delta^k_i = N$, as you surmise.



        Also $delta^i_i delta^j_j delta^k_k = displaystyle sum_{i=1}^N displaystyle sum_{j=1}^N displaystyle sum_{k=1}^N delta^i_i delta^j_j delta^k_k = N^3$, as you suspected.



        Finally, $delta^j_i delta^k_j = delta^k_i$ but this is not a scalar quantity ($neq 1$). Instead it has a separate value for each index $i$ and $k$. You can think of it as being represented by a matrix whose $(i,k)$th entry is $1$ if $i=k$ and $0$ otherwise (identity matrix in $N$ dimensions).



        This is a bit of a simplification of the whole picture but the essence of the machinery is here.






        share|cite|improve this answer












        It looks to me like you're inherently using the Einstein summation convention, familiar in general relativity. For the purposes of this question, this means that any indices seen twice are summed over.



        When you say $delta^i_j delta^j_i = N$, for example, this implicitly means $displaystyle sum_{i=1}^N displaystyle sum_{j=1}^N delta^i_j delta^j_i = N$ which is indeed true since the sum is non-vanishing whenever $i=j$ and this happens $N$ times.



        Similarly, $delta^i_j delta^j_k delta^k_i = displaystyle sum_{i=1}^N displaystyle sum_{j=1}^N displaystyle sum_{k=1}^N delta^i_j delta^j_k delta^k_i = N$, as you surmise.



        Also $delta^i_i delta^j_j delta^k_k = displaystyle sum_{i=1}^N displaystyle sum_{j=1}^N displaystyle sum_{k=1}^N delta^i_i delta^j_j delta^k_k = N^3$, as you suspected.



        Finally, $delta^j_i delta^k_j = delta^k_i$ but this is not a scalar quantity ($neq 1$). Instead it has a separate value for each index $i$ and $k$. You can think of it as being represented by a matrix whose $(i,k)$th entry is $1$ if $i=k$ and $0$ otherwise (identity matrix in $N$ dimensions).



        This is a bit of a simplification of the whole picture but the essence of the machinery is here.







        share|cite|improve this answer












        share|cite|improve this answer



        share|cite|improve this answer










        answered Sep 26 at 23:36









        hexomino

        1,07638




        1,07638






























            draft saved

            draft discarded




















































            Thanks for contributing an answer to Mathematics Stack Exchange!


            • Please be sure to answer the question. Provide details and share your research!

            But avoid



            • Asking for help, clarification, or responding to other answers.

            • Making statements based on opinion; back them up with references or personal experience.


            Use MathJax to format equations. MathJax reference.


            To learn more, see our tips on writing great answers.





            Some of your past answers have not been well-received, and you're in danger of being blocked from answering.


            Please pay close attention to the following guidance:


            • Please be sure to answer the question. Provide details and share your research!

            But avoid



            • Asking for help, clarification, or responding to other answers.

            • Making statements based on opinion; back them up with references or personal experience.


            To learn more, see our tips on writing great answers.




            draft saved


            draft discarded














            StackExchange.ready(
            function () {
            StackExchange.openid.initPostLogin('.new-post-login', 'https%3a%2f%2fmath.stackexchange.com%2fquestions%2f2930727%2fkronecker-delta-expressions%23new-answer', 'question_page');
            }
            );

            Post as a guest















            Required, but never shown





















































            Required, but never shown














            Required, but never shown












            Required, but never shown







            Required, but never shown

































            Required, but never shown














            Required, but never shown












            Required, but never shown







            Required, but never shown







            Popular posts from this blog

            Quarter-circle Tiles

            build a pushdown automaton that recognizes the reverse language of a given pushdown automaton?

            Mont Emei